LSAT and Law School Admissions Forum

Get expert LSAT preparation and law school admissions advice from PowerScore Test Preparation.

 Administrator
PowerScore Staff
  • PowerScore Staff
  • Posts: 8916
  • Joined: Feb 02, 2011
|
#85571
Complete Question Explanation

The correct answer choice is (C).

Answer choice (A):

Answer choice (B):

Answer choice (C): This is the correct answer choice.

Answer choice (D):

Answer choice (E):

This explanation is still in progress. Please post any questions below!
User avatar
 lsatquestions
  • Posts: 66
  • Joined: Nov 08, 2021
|
#95607
I eliminated B and E here but was stuck with the rest as contenders. I see that C is the correct answer now because the author likely believes that major eruptions may 'indirectly' cause cold summers, but not directly. Can you explain where in the passage we can find evidence to eliminate A and D though? For A is it because we don't know if a 1/2 degree drop can cause a feedback loop where temperatures ultimately drop more than a degree? For D is it because minor eruptions do not cause the feedback loop, and thus the difference is magnified when major eruptions do cause a feedback loop?
 Rachael Wilkenfeld
PowerScore Staff
  • PowerScore Staff
  • Posts: 1358
  • Joined: Dec 15, 2011
|
#95620
I think you are asking the wrong questions here, lsatquestions. The question is not where is an answer choice disproven in the passage, but where the correct answer choice can be proven. That is, if you can't find support for an answer choice in this question type from the information in the passage, that answer choice is incorrect. So for answer choice (A), we only have examples of a temperature drop of 1/2 a degree centigrade. We can't give any support for the assertion that there's a full degree drop. For answer choice (D), we also don't have support for that assertion from the passage. We know that the major and minor eruptions are closer than predicted. However, the information on the feedback loop could be referencing temperature drops from either major or minor eruptions. We don't know, and we certainly don't have enough to state that they are the same aside from a matter of degree.

For these question types, you want to go in with the mindset that the passage must prove the answer choice. If you don't have information on an answer choice either way, it's wrong and should be eliminated.

Hope that helps!

Get the most out of your LSAT Prep Plus subscription.

Analyze and track your performance with our Testing and Analytics Package.